PLEASE HELP

The graph of g(x) is shown.

The graph has...

#1.
a. the same horizontal asymptote as function g
b. a horizontal asymptote at y = 5
c. a horizontal asymptote at y = 8

#2. The graph has...

a. the same vertical asymptote as function g
b. a vertical asymptote at x = -7
c. a vertical asymptote at x = -5
d. a vertical asymptote at x = 3

PLEASE HELPThe Graph Of G(x) Is Shown. The Graph Has... #1. A. The Same Horizontal Asymptote As Function

Answers

Answer 1

On the given graph, we can see that we have:

Horizontal asymptote at y = 3Vertical asymptote at x = -2

The correct option would be a in both cases.

What can we conclude about the graph of g(x)?

We want to study the asymptotes of the graph

If we look at the graph, we can see that the horizontal asymptote (denoted by the horizontal dashed line) is at y = 3.

the graph has a horizontal asymptote (or two actually) at y = 3.

For the vertical asymptote we just look at the vertical dashed line, it is at x = -2

Notice that none of these coincides with the options, so the only options that can be correct (depending on the graph of g(x), which is not shown) are option "a" in both cases.

If you want to learn more about asymptotes:

https://brainly.com/question/1851758

#SPJ1


Related Questions

A population of amoebas in a petri dish will triple in size every hour. At the start of an experiment the population is 800. The function y equals 800 times 3 to the power of x , where x is the number of hours, models the population growth. How many amoebas are in the petri dish after 9 hours?

Answers

Answer:

15,746,400

Step-by-step explanation:

This is an example of an exponential function.

1. The function is: [tex]800 * 3^{x}[/tex]

2. We can plug the number of hours into the equation.

3. population = 800 * [tex]3^{9}[/tex]

4. population = 15,746,400

Please answer this question fast (Lines and angles ch...class9)

Answers

The angle of y in the triangle is 30 degrees.

How to find angles in a triangle?

∠O = 180 - 30 - 70 (angles in a triangle)

∠O = 180 - 100(angles in a triangle)

∠O = 80°

Using vertically opposite angle principle and sum of angle in a triangle,

80 + y + 70 = 180

180 - 150 = y

y = 30 degrees

learn more on angles here: https://brainly.com/question/17738167

#SPJ1

X = square root=
Help me please thanks

Answers

The value of x will be equal to 33.94 units.

What is trigonometry?

Trigonometry is the branch of mathematics that set up a relationship between the sides and angle of the right-angle triangles.

Given that:-

Given that the radius of the circle is 12 units which are PO = 12 and OR = 12, QR = x , ∠O = 60 So ∠Q = 30.

First, we will calculate the Length OQ by angle property in triangle OQR.

[tex]Sin 30=\dfrac{Perpendicular}{Hypotenuse}[/tex]

[tex]Sin 30 = \dfrac{12}{OQ-12}[/tex]

[tex]\dfrac{1}{2}=\dfrac{12}{OQ-12}[/tex]

OQ - 12 = 24

OQ = 36

Now applying the Pythagorean theorem in the triangle OQR.

H² = P² + B²

36² = 12² + x²

x² = 1296 - 144

x = √1152

x = 33.94

Therefore the value of  x will be equal to 33.94 units.

To know more about Trigonometry follow

https://brainly.com/question/24349828

#SPJ1

aimee has 20 apples and wants to have the same amount of apples in each bag but ends up with more than one bag of apples and each bag to contain more than one apple
work out all the possibilities for all of the number of apples that she could use for each bag

Answers

Answer:

There are 4 possibilities: 2, 4, 5 or 10 apples in each bag.

===========

Let the number of apples in each bag is x, then the number of bags is 20/x.

Since the number of apples in each bag is greater than one and number of bags is greater than one, we have the following conditions:

x > 1 , 20/x > 1 ⇒ x < 20,x is factor of 20.

The factors of 20 between 1 and 20 are:

2, 4, 5, 10

The possibilities are:

If x = 2, then the number of bags is 20/2 = 10;If x = 4, then the number of bags is 20/4 = 5;If x = 5, then the number of bags is 20/5 = 4;If x = 10, then the number of bags is 20/10 = 2.

If an artist combines 6 L of 33% by volume solution and 4 L of 60% by volume solution, what's the percent by volume of the resulting solution

Answers

The percent by volume of the resulting solution is 10L by 93 percent

How to determine the volume

The resulting solution = sum of the volumes of both solutions

Resulting solution = 6L × 33 percent + 4L × 60 percent

Resulting solution = 10 L × 93 percent

This is so because both solutions add up to form the resulting solution

Thus, the percent by volume of the resulting solution is 10L by 93 percent

Learn more about equivalent ratio here:

https://brainly.com/question/2328454

#SPJ1

Please help! Consider the sequence {20, 17, 14, 11, 8, 5, 2...}.

Answers

Answer:

c.) 38

Step-by-step explanation:

[tex]\sum\limits_{n=3}^6 a_n[/tex]   means "summation of the all the elements starting from the 3rd element to the 6th element".

The 3rd element in the series is 14, and the 6th element is 5; we have to add these and all the elements between them together.

∴    [tex]\sum\limits_{n=3}^6 a_n[/tex]    =  14 + 11 + 8 + 5

                   = 38

Find the probability. Leave your answer in simplest fraction form.
You roll a six-sided die. The die shows an even number or a number
greater than three.

Answers

Answer:

5/6

Step-by-step explanation:

The numbers on a six-sided die are as shown:

1, 2, 3, 4, 5, 6

The even numbers are:

2, 4, 6

The numbers greater than 3 are:

3, 4, 5, 6

Both lists together are:

2, 3, 4, 5, 6

Because 5 out of 6 numbers satisfy these conditions, the probability of satisfying these conditions is 5/6.

The answer to this is 5,6

Which statement regarding the diagram is true W, X,Z,Y

Answers

The statement that's true about the triangle is that WXY + YXZ = 180°.

How to illustrate the information?

It should be noted that in the question, the options are related to a linear pair.

According to the linear pair, when a line cuts another line at a point, then the sum of the adjacent angles formed at the point will be 180°.

Therefore, WXY + YXZ = 180°.

The correct option is C.

The complete question is:

Which statement regarding the diagram is true?

m∠WXY = m∠YXZ

m∠WXY < m∠YZX

m∠WXY + m∠YXZ = 180°

m∠WXY + m∠XYZ = 180°

Learn more about triangles on:

brainly.com/question/4001545

#SPJ1

Substracting 3x2+4x from 7x2+x+9 results in a polynomial. After subtracting 4x2-3x from this polynomial, the difference is ?

Answers

Answer:

9

Step-by-step explanation:

So first step is to subtract [tex]3x^2+4x[/tex] from [tex]7x^2+x+9[/tex]. In setting this up you get the following expression

[tex](7x^2+x+9)-(3x^2+4x)[/tex]

Distribute the negative

[tex]7x^2+x+9-3x^2-4x[/tex]

Group like terms

[tex](7x^2-3x^2)+(x-4x)+9[/tex]

Simplify:

[tex]4x^2-3x+9[/tex]

Now subtract [tex]4x^2-3x[/tex]. In setting this up you get the following expression

[tex](4x^2-3x+9)-(4x^2-3x)[/tex]

Distribute the negative:

[tex]4x^2-3x+9-4x^2+3x[/tex]

Group like terms

[tex](4x^2-4x^2) + (-3x+3x) + 9[/tex]

Simplify:

[tex]9[/tex]

Marta believes that the equation of the line of best fit for the scatterplot below is -
. Which statement best summarizes why Marta is likely incorrect?

Answers

Marta’s equation has a positive y-intercept, but the scatterplot shows a negative correlation.

What is a scatter plot?

The scatter plot is a manner in which data is presented as dots on a cartesian axes, The line of best fit is a description of the data that is presented in the scatter plot.

Hence, Marta is incorrect because Marta’s equation has a positive y-intercept, but the scatterplot shows a negative correlation.

Missing parts;

Marta believes that the equation of the line of best fit for the scatterplot below is y=-5/9x+23/9. Which statement best summarizes why Marta is likely incorrect?

Marta’s equation has a positive y-intercept, but the scatterplot suggests a negative y-intercept.

Marta’s equation has a positive y-intercept, but the scatterplot shows a negative correlation.

Marta’s equation has a negative slope, but the scatterplot suggests a negative y-intercept.

Marta’s equation has a negative slope, but the scatterplot shows a positive correlation.

Learn more about scatter plot:https://brainly.com/question/13984412

#SPJ1

seperate 90 into two parts so that one part is four times the other

Answers

Answer:

18 and 72

Step-by-step explanation:

the smaller part can be assigned as x while the larger will be 4x. Both numbers need to add up to 90, giving the equation: 4x+x = 90

Solve:

4x+x = 90

5x = 90

x = 18

4x = 72

Solve the equation for x.
6-√4+3x = 2

Answers

I think the answered is ×=20

6-2+3x=2
4+3x=2
3x=2-4
3x=-2
X=-2/3

A vector has a magnitude of 8 in the 95 degree direction. what are the horizontal and vertical components?

Answers

Vertical component = 8 x sin(95) = 7.970 (4sf)
Horizontal component = 8 x cos(95) = -0.6972 (4sf)

There are two numbers. One number is twice the other number. The difference of the smaller number and half the larger number is 20.

An equation created to find the smaller number will have

Answers

Step-by-step explanation:

There cannot be such a question because the difference of half of the big number and the small number will automatically be zero, but we can find it with the equation I circled on the paper I gave you. achievements

give the meaning of each expression.
16^5
y^4
7a^3

Answers

Answer:

16 to the power of 5

y to the power of 4

(7a) to the power of 3 or 7 times a to the power of 3

What is the area of the given figure?
10 in.
O 192 units²
O
92.9 units²
O 101.8 units²
O167.2 units2
O
176.7 units2
6 in.
10 in.
9 in.

Answers

[tex]\huge\boxed{192\ \text{in}^2}[/tex]

There are three parts to this figure: a rectangle and two triangles that are congruent.

We'll add together the area for each to get the total area.

We'll start by finding the area of the rectangle. We don't know its length, so we need to find the bases of the triangles and add them together.

We know that [tex]a^2+b^2=c^2[/tex]. Substitute and solve for [tex]a[/tex]:

[tex]\begin{aligned}a^2+6^2&=10^2\\a^2+36&=100\\a^2+36-36&=100-36\\a^2&=64\\\sqrt{a^2}&=\sqrt{64}\\a&=8\end{aligned}[/tex]

Now, double this to get the total length of the rectangle, which is [tex]16[/tex] inches.

The area of the rectangle is equal to its length times its height:

[tex]16\cdot9=\underline{144}[/tex]

Now, we'll find the area of one of the triangles and double it since they're congruent.

The area of a triangle is one-half of its base times its height, which we then double.

[tex]2\left(\frac{1}{2}\cdot b\cdot h\right)[/tex]

The [tex]2[/tex] and the [tex]\frac{1}{2}[/tex] cancel each other out.

[tex]b\cdot h[/tex]

Substitute and solve:

[tex]8\cdot6=\underline{48}[/tex]

Finally, add the rectangle's area to the two triangles' area.

[tex]144+48=\boxed{192}[/tex]

Which ordered pairs make the equation true? 3x+2y=−7 Select each correct answer. (3, −8)
(−3, 1)
(−2, −1)
(1, −4)

Answers

SOLVING

[tex]\Large\maltese\underline{\textsf{A. What is Asked}}[/tex]

Which ordered pairs make the equation [tex]\bf{3x+2y=-7}[/tex] true? Select all that apply. 3 options are given

[tex]\Large\maltese\underline{\textsf{B. This problem has been solved!}}[/tex]

We can tell which ordered pairs make equations true by plugging in the coordinates.

[tex]\mathbb{ORDERED\;PAIR\;NUMBER\;ONE}[/tex]

[tex]\bf{3(3)+2(-8)=-7}[/tex] | simplify

[tex]\bf{9-16=-7}[/tex] |simplify

[tex]\bf{-7\equiv-7}[/tex] | this one checks

[tex]\mathbb{ORDERED\;PAIR\;NUMBER\;TWO}[/tex]

[tex]\bf{3(-2)+2(-1)=-7}[/tex] | simplify

[tex]\bf{-6-2=-7}[/tex] | simplify

[tex]\bf{-8\neq-7}[/tex] | this one does not check

[tex]\mathbb{ORDERED\;PAIR\;NUMBER\;THREE}[/tex]

[tex]\bf{3(1)+2(-4)=-7}[/tex] | simplify

[tex]\bf{3-8=-7}[/tex] | simplify

[tex]\bf{-5\ne-7}[/tex] | this one does not check

[tex]\rule{300}{1.7}[/tex]

[tex]\bf{Result:}[/tex]

              [tex]\bf{=The\;First\;Option[/tex]

[tex]\boxed{\bf{aesthetic \not101}}[/tex]

For which value of k is the value of k(k-2)(k+1) negative

Answers

Answer:

[tex](-\infty, -1)\ \cup\ (0, 2)[/tex]

Step-by-step explanation:

So if you expand out the two binomials (k-2)(k+1), you'll get: [tex]k^2+k-2k-2[/tex]. which simplifies to: [tex]k^2-k-2[/tex]. Multiplying this by the k gives you: [tex]k^3-k^2-2k[/tex]. As you can see the degree is odd, this means that this polynomial will have two opposite end behaviors. And as you can see the leading coefficient is positive, meaning that this function will go towards positive infinity as k goes towards positive infinity. Also we if look at the original equation given, it's in factored form, with the zeroes as k=0, k=2, and k=-1. So given this we can draw a simple graph to see when the value of the equation is negative. If you look at the graph I drew you'll see that it's negative from (-infinity, -1) and then negative from (0, 2)

Determine if each table below represents a linear function, quadratic function, or neither.

Answers

The table below is a quadratic function.

What is a quadratic function?

A quadratic function is an Algebraic function with the power of its variable as 2.

This function normally has three algebraic terms, the x-square term, the x-term and the constant term.

Analysis:

The X-column values increases from -2 to 1, while the f(x) column which is the y-column increase from -9, until it gets to 1, where the value falls to -5.

A typical behavior of an inverted-v curve which is a quadratic curve.

If it were a linear function, as x values increase, y value would either keep increasing or decreasing, it does not change its orientation.

Learn more about quadratic curves: brainly.com/question/1214333

#SPJ1

URGENT!!!WORTH 27 POINTS!!!

Doug can download new songs for $1.19 each. Write an equation to show how many songs he can download for $12.00.

A. 12x=1.19
B. 12+x=1.19
C. 1.19+x=12
D. 1.19x=12

Answers

Answer:

1.19x = 12

Step-by-step explanation:

To write an equation, we multiply the cost of each song by the number of songs, x

1.19 * x

This is equal to the total amount he is allowed to spend

1.19x = 12

Answer:

D

Step-by-step explanation:

If Doug can download new songs for $1.19 each, then he will be able to download 12 songs.

A study on the latest fad diet claimed that the amounts of weight lost by all people on this diet had a mean of 23.4 pounds and a standard deviation of 6.8 pounds.
Step 2 of 2 : If a sampling distribution is created using samples of the amounts of weight lost by 63 people on this diet, what would be the standard deviation of the sampling distribution of sample means? Round to two decimal places, if necessary.

Answers

Using the Central Limit Theorem, the standard deviation of the sampling distribution of sample means would be of 0.86.

What does the Central Limit Theorem state?

It states that the standard deviation of the sampling distribution of sample means is given by:

[tex]s = \frac{\sigma}{\sqrt{n}}[/tex]

In which:

[tex]\sigma[/tex] is the standard deviation of the population.n is the sample size.

The parameters for this problem are given as follows:

[tex]\sigma = 6.8, n = 63[/tex].

Hence:

[tex]s = \frac{\sigma}{\sqrt{n}}[/tex]

[tex]s = \frac{6.8}{\sqrt{63}}[/tex]

s = 0.86.

More can be learned about the Central Limit Theorem at https://brainly.com/question/16695444

#SPJ1

Deepak wrote out the steps to his solution of the equation startfraction 5 over 2 minus 3 x minus 5 plus 4 x equals negative startfraction 7 over 4 endfraction – 3x – 5 4x = –.

Answers

The solution is x=3/4

How can we solve given equation?

First, we will solve like terms. Then shift constant to other side and keep x on the same side to get the value of x.

We can solve given equation as shown below:

5/2-3x-5+4x=-7/4

(5-10)/2+x=-7/4

-5/2+x=-7/4

x=5/2-7/4

x= (10-7)/4

x=3/4

Hence, the solution is x=3/4.

Learn more about Linear equations here:

https://brainly.com/question/43297

#SPJ4

how many real solutions does this system of equations have?
y=x2+1
y=x

A. 1
B. 2
C. 3
D. 0

Answers

The system of equations has (d) 0 real solutions

How to determine the number of real solutions?

The system of equations is given as:

y = x^2+1

y=x

Substitute y=x in y = x^2+1

x = x^2+1

This gives

x^2 - x + 1 = 0

Calculate the discriminant using:

d = b^2 - 4ac

So, we have:

d = (-1)^2 - 4 * 1 * 1

Evaluate

d = -3

Because the discriminant is negative, the equation has no real solution

Hence, the system of equations has (d) 0 real solutions

Read more about system of equations at:

https://brainly.com/question/12895249

#SPJ1

Using mod, find the remainder of 3^51 when divided by 7. Please show steps on how to use modulu, am a bit confused.

Answers

Answer:

[tex]6[/tex]

Step-by-step explanation:

The gist of modular arithmetic in a nutshell: the numbers [tex]a[/tex] and [tex]b[/tex] are considered to be congruent by their modulus [tex]m[/tex] if [tex]m[/tex] is a divisor of their difference.

In mathematics: [tex]a \equiv_{m} b \Leftrightarrow (a - b) \vdots m[/tex]

Exemplifying this: [tex]6 \equiv_{7} -1[/tex] because [tex]6 - (-1) = 6 + 1 = 7[/tex], [tex]7 \vdots 7[/tex].

Let us have following equivalences: [tex]a \equiv_{m} b[/tex] and [tex]c \equiv_{m} d[/tex], then:  [tex](a - b) \vdots m[/tex] and [tex](c - d) \vdots m[/tex] by definition.

Properties:

1. [tex]a + c \equiv_{m} b + d \Leftrightarrow ((a + c) - (b + d)) \vdots m \Leftrightarrow (a + c - b - d) \vdots m \Leftrightarrow ((a - b) + (c - d)) \vdots m[/tex].

2. [tex]a - c \equiv_{m} b - d \Leftrightarrow ((a - c) - (b - d)) \vdots m \Leftrightarrow (a - c - b + d) \vdots m \Leftrightarrow ((a - b) - (c - d)) \vdots m[/tex].

3. [tex]ac \equiv_{m} bd \Leftrightarrow (ac - bd) \vdots m \Leftrightarrow (ac - bc - bd + bc) \vdots m \Leftrightarrow (c(a - b) + b(c - d)) \vdots m[/tex].

4. What if we have [tex]a \equiv_{m} b[/tex] twice? If we abide by property 3, we can come to the conclusion that [tex]a^2 \equiv_m b^2[/tex]. It is fair enough that there is room for the equivalence [tex]a^n \equiv_{m} b^n[/tex].

[tex]3^{51} = (3^3)^\frac{51}{3} = 27^{17} \equiv_{7} (-1)^{17} \equiv_{7} -1 \equiv_{7} 6[/tex].

We used property 4.

Keep in mind that any remainder cannot be a negative number.

Therefore, the remainder equals [tex]6[/tex].

3 Quick algebra 1 Questions for 50 points!

For (lovetthannah9) or anyone who knows the answer! :)

Answers

Answer:

Step-by-step explanation:

6. The rate of change is essentially the slope. But in other words, it's how much the y changes as x increases by 1. If this rate of change is 0, that means the y-value is constant, while the x can be any real number. It can be generally given in the formula: y=b, which comes from simplifying the slope-intercept form: y=(0)x+b = b. You can also derive the same thing for a point-slope form: y-a = 0(x-b)=0 so y=a. In both cases, y is equal to some constant value. So graphing this makes a horizontal line.

7. To define a parallel line, you need the same slope, and a different y-intercept, which is under the assumption that the line has a slope that is definable. If the slope is definable, you'll have the equation: y=mx+b, where m is the same as the other line. Then you plug in the known values as x and y, to solve for b, which is the y-intercept. Now if the line is a vertical line, a parallel line can be defined as: x=a, where a doesn't equal the constant, the other vertical line is equal to. To make it pass through some point (c, d). Then you would simply set x=c, since the x is the only thing that matters, since the y-value is all real numbers, so it will eventually have the y-value d somewhere on the line.

8. To define a perpendicular line, you get the reciprocal of the slope and change the sign. So if the slope is: [tex]\frac{a}{b}[/tex] then it becomes [tex]-\frac{b}{a}[/tex] and vice versa depending on the sign. In this example, let's just say m=a/b, then the perpendicular slope would be -b/a. You can then plug this into the slope-intercept formula: [tex]y=-\frac{b}{a}+c[/tex] where c is the y-intercept. Then you plug in the known point as (x, y) and solve for c, the y-intercept. Now let's say for example that you have a vertical line: x=a. In this case you can see the slope as: [tex]\frac{1}{0}[/tex][tex]\frac{y_2-y_1}{a-a}[/tex]. Of course this isn't definable, but if you take the reciprocal you get something that is: [tex]-\frac{0}{y_2-y_1}[/tex] which will always evaluate to 0. This means you get a horizontal line, since the slope is 0. This means if you have a vertical line, any horizontal line should be perpendicular, which makes sense, since it should form a 90 degree angle when they intersect, because they're straight lines. To make sure that horizontal line passes through the point (b, c), you simply set the y equal to c. So y=c, will pass through (b, c), since the c is constant, and the x can equal anything so somewhere on the line it will intersect (b, c). But let's say we had a horizontal line, the reciprocal can be defined as: [tex]\frac{a-a}{x_2-x_1}[/tex] where a is the y constant. If you take the reciprocal, then you have a-a in the denominator, which gives you an undefined slope, because the perpendicular line to a horizontal line, is a vertical line. To ensure this perpendicular line passes through the point (b, c). You simply set x equal to b. so that x=b.

#6

y=mx+c

if m is 0

y=c is the Equation

Hence line is parallel to x axis

#7

parallel lines have equal slopes

so we can use the point slope form of line to find the equation

y-y_1=m(x-x_1)

#2

Same process like no 7 but

slope of perpendicular line is negative reciprocal of given line's slope

Which function has zeros at x = -2 and x = 5?
O f(x) = x2 + 2x - 10
• f(x) = x2 - 2x - 10
O f(x) = X2 + 3x - 10
® f(x) = x2 - 3x - 10

Answers

Answer:

f(x)=x²-3x-10

Step-by-step explanation:

[tex]f(x) = x {}^{2} - 3x - 10 \\ to \: find \: x \: intercept \:o r \: zero \: substitute \: f(x) = 0\: \\ 0 = x {}^{2} - 3x - 10 \\ x {}^{2} - 3x - 10 = 0 \\ x {}^{2} + 2x - 5x - 10 = 0 \\ x(x + 2) - 5x - 10 = 0 \\ x(x + 2) - 5(x + 2) = 0 \\ (x + 2).(x - 5) = 0 \\ x + 2 = 0 \\ x - 5 = 0 \\ x = - 2 \\ x = 5[/tex]

therefore the zeros of the equation are x₁=-2,x₂=5

For the following exercises, solve each inequality and write the solution in interval notation.
31. | 3x − 4 | ≤ 8

Answers

Answer:

The solution set in interval form is [tex]$\left[\frac{-4}{3}, 4\right]$[/tex].

Step-by-step explanation:

It is given in the question an inequality as [tex]$|3 x-4| \leq 8$[/tex].

It is required to determine the solution of the inequality.

To determine the solution of the inequality, solve the inequality [tex]$3 x-4 \leq 8$[/tex] and, [tex]$-8 \leq 3 x-4$[/tex].

Step 1 of 2

Solve the inequality [tex]$3 x-4 \leq 8$[/tex]

[tex]$$\begin{aligned}&3 x-4 \leq 8 \\&3 x-4+4 \leq 8+4 \\&3 x \leq 12 \\&x \leq 4\end{aligned}$$[/tex]

Solve the inequality [tex]$-8 \leq 3 x-4$[/tex].

[tex]$$\begin{aligned}&-8+4 \leq 3 x-4+4 \\&-4 \leq 3 x \\&-\frac{4}{3} \leq x \\&x \geq-\frac{4}{3}\end{aligned}$$[/tex]

Step 2 of 2

The common solution from the above two solutions is x less than 4 and [tex]$x \geq-\frac{4}{3}$[/tex]. The solution set in terms of interval is [tex]$\left[\frac{-4}{3}, 4\right]$[/tex].

rewrite 4 1/2=2
please

Answers

The logarithmic expression of 4^(1/2) = 2 is [tex]\log_4(2) = \frac 12[/tex]

How to rewrite the expression?

The expression is given as:

4^(1/2) = 2

Take the logarithm of both sides

log(4^(1/2)) = log(2)

Apply the change of base rule

1/2log(4) = log(2)

Divide both sides by log(4)

1/2 = log(2)/log(4)

Change the base

[tex]\frac 12 =\log_4(2)[/tex]

Rewrite as:

[tex]\log_4(2) = \frac 12[/tex]

Hence, the logarithmic expression of 4^(1/2) = 2 is [tex]\log_4(2) = \frac 12[/tex]

Read more about logarithmic expression at:

https://brainly.com/question/24211708

#SPJ1

Which describes the inverse operations used after the distributive property?
addition then division
subtraction then division
multiplication then subtraction
division then addition

Answers

The inverse operations used after the distributive property is B. subtraction then division.

How to illustrate the information?

The equation given goes thus:

5(x + 6) = 50.

5x + 30 = 50

5x = 20.

x = 4

Therefore, the inverse operations used after the distributive property is subtraction then division.

Learn more about distributive property on:

brainly.com/question/1780105

#SPJ1

Using the following image, solve for SR. Look at the image closely.
What would SR be? Giving Brainly!

Answers

Step-by-step explanation:

I assume

SR = 2x + 23

RQ = x + 21

if that is true, then the situation is completely simple :

14 = (2x + 23) + (x + 21) = 3x + 44

3x = -30

x = -10

SR = 2×-10 + 23 = -20 + 23 = 3

RQ = -10 + 21 = 11

[tex]\quad \huge \quad \quad \boxed{ \tt \:Answer }[/tex]

[tex]\qquad \tt \rightarrow \:x = -10 [/tex]

____________________________________

[tex] \large \tt Solution \: : [/tex]

[tex] \qquad \tt \rightarrow \: SR + RQ = SQ[/tex]

[tex]\qquad \tt \rightarrow \: 2x + 23 + x + 21 = 14[/tex]

[tex]\qquad \tt \rightarrow \: 3x + 44 = 14[/tex]

[tex]\qquad \tt \rightarrow \: 3x = 14 - 44[/tex]

[tex]\qquad \tt \rightarrow \: 3x = - 30[/tex]

[tex]\qquad \tt \rightarrow \: x = - 10[/tex]

Now,

[tex] \qquad \tt \rightarrow \: SR = 2x + 23 [/tex]

[tex] \qquad \tt \rightarrow \: SR = 2(-10) + 23 [/tex]

[tex] \qquad \tt \rightarrow \: SR =-20+ 23 [/tex]

[tex] \qquad \tt \rightarrow \: SR = 3 \:\: units [/tex]

Answered by : ❝ AǫᴜᴀWɪᴢ ❞

Other Questions
Identity the x-intercept of the liner equation 3x + y -9=0 What is the leastcommon multiple of 12 and 16? Complete the sentences with words that are punctuated and spelled correctly.Natasha was especially interested in dinosaurs; however, Jorge wanted to check out the huge collection of precious gems.Natasha gazed into a replica of a cooking fire and began to imagine sounds: a crying baby, the persistent click of stone on stone involved in making scraping tools, and the thunderous cry of a mastodon. Prompt 1: Take the position of the colonist and explain how you might feel in this situation and why?Prompt 2: Now take the position of the Native Americans and explain how you might feel in this situation and why?Prompt 3: Finally, be objective and write how you think the Native Americans and settlers could have both existed peacefully together HELP ME PLEASEEE 20 POINTSSS!!!!Fighter pilots the maximum height, h, of a fighter pilot is 77 inches. Write this as an inequality.h 70h 77h 77h 70 The surface area of a cone with radius r units and slant height s units is shown below:30 points Surface area = 3.14(rs + r2)Part A: If r = 3 units and s = 5 units, write an expression that can be used to calculate the surface area of the cone. (4 points)Part B: What is the surface area of the cone? Show your work. (6 points) How does the author create an element of surprise in the passage?Select the correct answer. Which of the following models the context "A plumber who charges $50 for ahouse call and $85 per hour"? A start-up software company hires Kiera as its first human resource manager. Whenever a new position needs to be filled, Kiera sits down with the supervisor to gather facts that will be included in a realistic job preview. Kiera knows that the impact of job previews on candidates is limited, but she believes the effort is important anyway. Which statement best explains Kiera's decision to provide realistic job previews?A) The recruiter is less important than personnel policies for affecting a candidate's decision.B) Realism in the recruiter's message is the most researched aspect of recruiting.C) Kiera feels pressure to exaggerate the negative qualities of each job vacancy.D) Employees are highly sensitive to negative information.E) Giving realistic information is not expensive or difficult, so it is worth even a small benefit. Wha is contained in chromosomes give the right answer and you will be marked brainliest write down the factors that affect weight of a body. what happens when solid particles gain enegry When butane burns completely, only water and carbon dioxide gas are produced. If 11.6 g of butane and 40.0 L of oxygen at 22.0o C and 102 kPa react, what volume of carbon dioxide gas also at 22.0o C and 102 kPa can be collected over water. The vapour pressure of water at 22.0o C is 2.24 kPa. 2C4H10(g) + 13O2(g) --> 10H2O(g) + 8CO2(g) Solve for x. Triangle ABC is similar to Triangle FED Stars that have ejected a planetary nebula go on to become white dwarfs. supernovae. protostars. red giants. Three ropes A, B and C are tied together in one single knot K. (See figure.)If the tension in rope A is 65.3 N, then what is the tension in rope B? Compare the ratios using the models. One model has 16 sections and is labeled 7 to 16. The second model has 8 sections and is labeled 6 to 8.Which statements are true? Check all that apply.The model for the ratio 7 to 16 should show 7 cells shaded.The model for the ratio 6:8 should show 8 cells shaded.The ratio 7 to 16 is greater than 6:8.The ratio 6:8 is greater than the ratio 7 to 16 Please answer this question Someone please help me with this question?